Last visit was: 26 Apr 2024, 20:50 It is currently 26 Apr 2024, 20:50

Close
GMAT Club Daily Prep
Thank you for using the timer - this advanced tool can estimate your performance and suggest more practice questions. We have subscribed you to Daily Prep Questions via email.

Customized
for You

we will pick new questions that match your level based on your Timer History

Track
Your Progress

every week, we’ll send you an estimated GMAT score based on your performance

Practice
Pays

we will pick new questions that match your level based on your Timer History
Not interested in getting valuable practice questions and articles delivered to your email? No problem, unsubscribe here.
Close
Request Expert Reply
Confirm Cancel
SORT BY:
Kudos
Tags:
Show Tags
Hide Tags
User avatar
Senior Manager
Senior Manager
Joined: 25 Oct 2008
Posts: 361
Own Kudos [?]: 6508 [11]
Given Kudos: 100
Location: Kolkata,India
 Q44  V38
Send PM
avatar
Intern
Intern
Joined: 15 Mar 2008
Posts: 22
Own Kudos [?]: 52 [3]
Given Kudos: 0
Send PM
User avatar
Senior Manager
Senior Manager
Joined: 25 Oct 2008
Posts: 361
Own Kudos [?]: 6508 [1]
Given Kudos: 100
Location: Kolkata,India
 Q44  V38
Send PM
avatar
Intern
Intern
Joined: 28 Apr 2009
Posts: 18
Own Kudos [?]: 13 [0]
Given Kudos: 2
Send PM
Re: If the economy is weak, then prices remain constant although unemploym [#permalink]
Given the passage, I would go with C.

a) It is given in the question that the investment is not decreasing, and also that this is true :-). So investment cannot be decreasing.

b) the statement says. if x (weak economy) then y (constant prices) and z (rise in unemployment).. no evidence to prove that if z then y.

c) sounds OK compare to others. if the investment decreases, unemployment will raise. note that 'only' is given in cause for rise in unemployment.

So if it has to be x then y and z should hold true. But for z to hold true investement should decrease.

So the economy is weak only if the investement decreases.


d)Both cannot be in a either/or situation. Both of them occur at the same time.

e)statement Clearly states that the investement is not decreasing. So there is no chance that the unemployment is raising.

and the defence rests !!

alternate opinions ??
User avatar
Director
Director
Joined: 08 May 2009
Status:There is always something new !!
Affiliations: PMI,QAI Global,eXampleCG
Posts: 552
Own Kudos [?]: 589 [0]
Given Kudos: 10
Send PM
Re: If the economy is weak, then prices remain constant although unemploym [#permalink]
its clearly mentioned that investment is not decreasing.
Hence except A others are all possible scenarios.

A
Director
Director
Joined: 22 Mar 2013
Status:Everyone is a leader. Just stop listening to others.
Posts: 611
Own Kudos [?]: 4596 [0]
Given Kudos: 235
Location: India
GPA: 3.51
WE:Information Technology (Computer Software)
Send PM
Re: If the economy is weak, then prices remain [#permalink]
umeshpatil wrote:
If the economy is weak, then prices remain constant although unemployment rises. But unemployment rises only if investment decreases. Fortunately, investment is not decreasing.

If the statement above are true, then which one of following must be false.
(A) Either the economy is weak or investment is decreasing.
(B) If unemployment rises, then prices remain constant.
(C) The economy is weak only if investment decreases.
(D) Either the economy is weak or prices are remaining constant.
(E) Either unemployment is rising or the economy is not weak.

If you like this fresh LSAT CR question, +1 kudo is best way to say thanks :)


I am not able to filter out any option.
I have considered three test results for any condition with provided data.
1. True. (1)
2. Undecided (-)
3. False. (0)

Relation given to us :

(IF) IC is weak --> (THEN) Price constant (in spite of) unemployment rises.
(IF)Investment decrease ---> (THEN) unemployment increase.

Current status = investment is not decreasing

Test:
(A) Either the economy is weak or investment is decreasing.
Either the economy is weak = Can't decide (-)
Can we say that decrease in unemployment --->> strong economy ?? is reverse causality is allowed as per provided information ??
if yes then only my answer would be (No)

Investment is decreasing= No
(-) || (0) = ??

(B) If unemployment rises, then prices remain constant.
No relation provided between employment and prices = (-)

(C) The economy is weak only if investment decreases.
No relation can be deduced from provided relations = (-)

(D) Either the economy is weak or prices are remaining constant.
As per provided relationship these two are inter-related state thus they can not exist independently.
Yes economy is weak ---> Prices are constant = (1)
Prices are constant ----> Economy is weak = (-)
From provided information I can not deduce current state of any factor.

(E) Either unemployment is rising or the economy is not weak.
Unemployment rising = No because investment is rising. (0)
The economy is weak (-)

Can you share the OE.
avatar
Intern
Intern
Joined: 01 Mar 2016
Posts: 6
Own Kudos [?]: 3 [0]
Given Kudos: 9
Send PM
Re: If the economy is weak, then prices remain [#permalink]
umeshpatil wrote:
If the economy is weak, then prices remain constant although unemployment rises. But unemployment rises only if investment decreases. Fortunately, investment is not decreasing.

If the statement above are true, then which one of following must be false.
(A) Either the economy is weak or investment is decreasing.
(B) If unemployment rises, then prices remain constant.
(C) The economy is weak only if investment decreases.
(D) Either the economy is weak or prices are remaining constant.
(E) Either unemployment is rising or the economy is not weak.

If you like this fresh LSAT CR question, +1 kudo is best way to say thanks :)



A.If economy is weak -> price constant and unemployment rises ( unempl rises -> only if investment decreases)->investment is decreasing.
hence weak economy and investment decrease occurs together, which makes A false.

B. No relation btw unemploymen rise and prices true/false at same time
C. true
D. possible/not possible ( we are only given info that prices remains constant if eonomy is weak, if economy is not weak we dont know what will happen)
E. we dont know what will happen when economy is not weak
Intern
Intern
Joined: 09 Apr 2017
Posts: 48
Own Kudos [?]: 42 [0]
Given Kudos: 368
GPA: 3.99
Send PM
Re: If the economy is weak, then prices remain constant although unemploym [#permalink]
If I were to see this as a math relationship, as a Data sufficiency question.

Premise 1:
If Econ decrease (weaken), P is constant , Unemployment increase (strengthen)
This form the equation that Econ is inverse relationship with Unemployment with constant P
1/E = P X U => Equation 1

Premise 2:
If Unemployment increase (strengthen), Investment decrease (weaken)
This form the equation that Investment is inverse relationship with Unemployment with constant P
1/I = P X U => Equation 2

Premise 3: (Inferred)
Equation 1 & Equation 2:
1/EI = PU => Equation 3.

Conclusion:
From equation 3 we knows that E and I is inverse relation of U with constant P
1/EI = PU

A "Either the economy is weak or the investment is decreasing"
Must be True.
When E, or I decrease -> U increase.
As stated in conclusion (Equation 3) : E and I is inverse relation of U with constant P

B "If unemployment rises then prices remain constant
Many possibilties of E and I When U Increase, P constant since we do not know what is E or I.
Case 1: E decrease, I increase to a minimal amount (or no changes) that allows U to decrease, with constant P.
Case 2: I decrease, E increase to a minimal amount (or no changes) that allows U to decrease, with constant P.
Case 3: E decrease, I decrease and U decrease, with constant P.
Case 4: E decrease, I decrease and U decrease, with constant P.

C "The economy is weak only if investment decreases".
E should be determine by U since E has inverse relationship with U (not only if I)

D "Either the economy is weak or the prices are remaining constant".
Many possibilities of I and U

E "Either unemployment is rising or the economy is not weak".
Out of scope - U an E are of inverse relationship.
Senior Manager
Senior Manager
Joined: 10 Aug 2021
Posts: 374
Own Kudos [?]: 35 [0]
Given Kudos: 226
Send PM
Re: If the economy is weak, then prices remain [#permalink]
Hello expert,
Could you explain why A must be wrong and D could be correct? For me, they are the same in logic, so if A must be wrong, the same reason D must be wrong.
Either Or means the 2 things happen mutually exclusively, but passage tells us If the economy is weak (X), then prices remain constant (Y), so this means if X happens, then Y must happen together, so they cannot be mutually exclusive. So I think D also must be wrong.
Pls help, and thx in advance.
GMAT Club Bot
Re: If the economy is weak, then prices remain [#permalink]
Moderators:
GMAT Club Verbal Expert
6923 posts
GMAT Club Verbal Expert
238 posts
CR Forum Moderator
832 posts

Powered by phpBB © phpBB Group | Emoji artwork provided by EmojiOne